Using lengths a, b, and c in the right triangle shown, how are the trigonometric functions of defined?

Answers

Answer 1
We need to see the triangle to answer this question.
Answer 2

Answer:

take a pic.of question

Step-by-step explanation:

gotta see the triangle


Related Questions

If the slope between the points (-5,7) and (4,x) is 1, then
what is the value of x?

Answers

9514 1404 393

Answer:

  16

Step-by-step explanation:

The equation of a line with slope m=1 through the point (h, k) = (-5, 7) will be ...

  y -k = m(x -h)

  y -7 = x +5

  y = x +12

Then for x = 4, the value of y is ...

  y = 4 +12 = 16

The point is (4, x) = (4, 16), so x = 16.

Answer:

16

Step-by-step explanation:

Given that the slope of the line is 1 . And it passes through the points (-5,7) and (4,x) .We know that slope is difference of ordinate by difference of abssica . So that ,

[tex]\implies Slope =\dfrac{y_2-y_1}{x_2-x_1}\\\\\implies 1 =\dfrac{ 7-x}{-5-4} \\\\\implies 1 \times -9 = 7-x \\\\\implies -9 -7 = -x \\\\\implies \underline{\underline{ x = 16 }}[/tex]

Hence the required answer is 16 .

The “Chippy” brand of chips cost $2.30 per bag. The “Crunchy Goodness” chips cost $6.30 for 3 bags of chips. Which bag has a lower unit price?

Answers

Answer: The chippy brand

Step-by-step explanation: Since 3 bags for the crunchy goodness is 6.30 dollars, we don;t need to worry about that. SO, lets see how much money is 3 bags from the chippy brand.

2.30 x 3 = 6.90

Chippy: $6.90

Crunchy Goodness: $6.30

Now that we compared the Chip brans, its obvious that crunchy goodness is cheaper

Answer:

The crunchy goodness chips.

Step-by-step explanation:

If the crunchy goodness chips cost $6.30 for 3 bags and the chippy chips cost $2.30 for each bag, you do $2.30 times 3. That equals 6.90. So, $6.30 is less than $6.90 meaning that crunchy goodness chips have a lower price.

I Question 7 (2 of 13)
Find area of shaded region. Round to the nearest tenth.
50 cm
radius: 20 cm
80 cm
Enter your answer by clicking the bubbles.

Answers

Method:
Area of trapezium - Area of a half circle

Trapezium = 40(50+80) divided by 2

= 2600

Half circle = 1/2 x pi x 20^2

= 628.3185....
~ 628.3

2600 - 628.3 = 1971.7 cm^2

Hope this helps!!



HELP HELP HELP!!! NUMBER 3 PLEASE!!!

Answers

Answer:

see the attachment, answers there

Supersonics Records plans to market a new record for a comeback artist nationwide. Management believes that there is a 70% chance that the new record will be a hit and, therefore, a 30% chance that the new record will be a miss. However, before resources are committed to market this new record nationwide, management will ask Mr. Deejay, a well-known authority in the music scene, to predict whether the new record will be a hit or a miss. Mr. Deejay's past predictions were quite impressive-che correctly predicted a hit record 80% of the time and incorrectly predicted a hit record only 10% of the time. Let H = The new record is actually a hit M = The new record is actually a miss DH = Mr. Deejay predicts the new record will be a hit Therefore, P(H) = 0.70 P(M) = 0.30 P(DH/H) = 0.80 P(DH/M) = 0.10 Based on these information, what is P(H and DH)? (That is, what is the probability that the new record is actually a hit (H) AND Mr. Deejay predicts the new record will be a hit (DH)?)
A. 0.56
B. 0.59
C. 0.95
D. Not enough information given to answer this question
E. None of the above

Answers

Answer:

A. 0.56

Step-by-step explanation:

Conditional Probability

We use the conditional probability formula to solve this question. It is

[tex]P(B|A) = \frac{P(A \cap B)}{P(A)}[/tex]

In which

P(B|A) is the probability of event B happening, given that A happened.

[tex]P(A \cap B)[/tex] is the probability of both A and B happening.

P(A) is the probability of A happening.

In this question:

We want to find [tex]P(H \cap DH)[/tex].

We can relate the numbers using [tex]A = H, B = DH[/tex]. So

[tex]P(B|A) = \frac{P(A \cap B)}{P(A)}[/tex]

[tex]P(DH|H) = \frac{P(DH \cap H)}{P(H)}[/tex]

P(H) = 0.70 , P(DH/H) = 0.80

So

[tex]P(DH \cap H) = P(DH|H)*P(H) = 0.8*0.7 = 0.56[/tex]

The correct answer is given by option A.

The population of cats that are rescued every year is currently 50,000 and declines at a rate of 1.2% every year. This models

Answers

Answer:

Exponential decay

A mixture of nuts contains 1 cup of walnuts for every 3 cups of peanuts write a linear equation that represents the relationships between penuts,x, and walnuts,y, Graph the line

Answers

Answer:

surfaces curving inward, so it makes parallel light rays curve outward or diverge. That's why concave lenses are sometimes called diverging lenses. ... The distance from the center of the lens to the focal point is, again, the focal length of the lens.

Step-by-step explanation:

Find the distance between P and Q. P(-3, -4), Q(6, -7)

Answers

Answer:

Please find attached pdf

Step-by-step explanation:

Step-by-step explanation:

P=(-3,-4)=(X1,Y1)

Q=(6,-7)=(X2,Y2)

Pq=root under( x2-x1)^2+(y2-y1)^2

= root under {6-(-3)}^2+{-7-(-4)}^2

= {6+3}^2+{-7+4}^2

= 9^2+(-3)^2

= 81+9

= 90

= 3root under10 unit.#

You are at a restaurant and the check comes to a total of $20. If you want to leave a 20% tip, how much total money should you pay, to the nearest cent?

Answers

Answer:

the total tip should be $24.00

Which sum is equivalent to 3/5 + 1/6?
A 6/10+2/10

B 12/30+5/30

C 18/30+5/30

D 3/11+4/11

Answers

Answer:

C 18/30+5/30

Step-by-step explanation:

Fractions scare me but I try

First we need to find the sum of 3/5 and 1/6

To do that we turn the denominators into common denominators.

[tex] \frac{18}{30} + \frac{5}{30} [/tex]

then we just add

[tex] \frac{18}{30} + \frac{5}{30} = \frac{23}{30} [/tex]

hope this makes sense

I WILL GIVE BRAINLYIST TO WHOEVER ANSWERS!!A toddler is playing with his toys in the bathtub. He is using a toy shaped like a rectangular prism, filling it with water to
the top, and filling his other toys to overflowing.
2 in.
Rectangular prism toy:
3 in.
5 in.
a. What is the volume of the rectangular prism toy?
in
b. What formula did you use to calculate your answer? V =
C. How many times will he need to fill the rectangular prism toy to overflow this cylindrical toy with a diameter of
5.2 inches and a height of 4 inches?
times

Answers

Answer:

A) 30 in³

B) V=lwh (or) V+5x3x2

C) 3 times

Step-by-step explanation:

A) 30in³

B) V=LWH (or) V+5x3x2

C) V=[tex]\pi[/tex]r²h = [tex]\pi[/tex](2.6²)(4) = 84.94867

85.94867÷30= 2.83, so he’d need to fill the rectangular prism 3 times.

Answer:

a. 30 in³

b. V = whl

c. 3 times

Step-by-step explanation:

a. v = whl

V = (3 in)(2 in)(5 in)

V= 30 in³

b. V = whl

c. Volume of the cylinder = pi(r²)h            r = d/2 = 5.2 / 2 = 2.6

Vc = pi (2.6²)(4)

Vc ≈ 84.94 in³

84.94 in³/30 in³ ≈ 3 times

You find that you receive on average about 3 pieces of junk mail per week. Find the probability that the number of pieces of junk mail you receive next week will be (a) none at all. (b) exactly four. (c) no more than two. (d) more than two g

Answers

Answer:

a) 0.0498 = 4.98% probability that the number of pieces of junk mail you receive next week will be none at all.

b) 0.1680 = 16.80% probability that the number of pieces of junk mail you receive next week will be exactly four.

c) 0.4232 = 42.32% probability that the number of pieces of junk mail you receive next week will be no more than two.

d) 0.5768 = 57.68% probability that the number of pieces of junk mail you receive next week will be more than two.

Step-by-step explanation:

We have only the mean, which means that we use the Poisson distribution to solve this question.

In a Poisson distribution, the probability that X represents the number of successes of a random variable is given by the following formula:

[tex]P(X = x) = \frac{e^{-\mu}*\mu^{x}}{(x)!}[/tex]

In which

x is the number of sucesses

e = 2.71828 is the Euler number

[tex]\mu[/tex] is the mean in the given interval.

You find that you receive on average about 3 pieces of junk mail per week.

This means that [tex]\mu = 3[/tex]

(a) none at all.

This is P(X = 0).

[tex]P(X = x) = \frac{e^{-\mu}*\mu^{x}}{(x)!}[/tex]

[tex]P(X = 0) = \frac{e^{-3}*3^{0}}{(0)!} = 0.0498[/tex]

0.0498 = 4.98% probability that the number of pieces of junk mail you receive next week will be none at all.

(b) exactly four.

This is P(X = 4).

[tex]P(X = x) = \frac{e^{-\mu}*\mu^{x}}{(x)!}[/tex]

[tex]P(X = 4) = \frac{e^{-3}*3^{4}}{(4)!} = 0.1680[/tex]

0.1680 = 16.80% probability that the number of pieces of junk mail you receive next week will be exactly four.

(c) no more than two.

This is:

[tex]P(X \leq 2) = P(X = 0) + P(X = 1) + P(X = 2)[/tex]

[tex]P(X = x) = \frac{e^{-\mu}*\mu^{x}}{(x)!}[/tex]

[tex]P(X = 0) = \frac{e^{-3}*3^{0}}{(0)!} = 0.0498[/tex]

[tex]P(X = 1) = \frac{e^{-3}*3^{1}}{(1)!} = 0.1494[/tex]

[tex]P(X = 2) = \frac{e^{-3}*3^{2}}{(2)!} = 0.224[/tex]

[tex]P(X \leq 2) = P(X = 0) + P(X = 1) + P(X = 2) = 0.0498 + 0.1494 + 0.224 = 0.4232[/tex]

0.4232 = 42.32% probability that the number of pieces of junk mail you receive next week will be no more than two.

(d) more than two

[tex]P(X > 2) = 1 - P(X \leq 2) = 1 - 0.4232 = 0.5768[/tex]

0.5768 = 57.68% probability that the number of pieces of junk mail you receive next week will be more than two.

What is a measurement 90 degrees called

Answers

It would be a right angle

Answer:

A right angle.

Step-by-step explanation:

Hope this helps! Have a great rest of your day!

A small bucket holds 75% as much as a large bucket.If the small holds 36 units,how much does the large hold?​

Answers

Answer:

48

Explanation:

36÷75=

0.48

4. Write a fraction that is a multiple of 4/5

Answers

8/10 is the fraction

Answer:

multiply by any number to create a multiple

Step-by-step explanation:

4/5 = 8/10 = 12/15 = 16/20...

00:00 A florist measures the lengths of roses. The lengths in inches of a dozen roses are given below. 13.5 13.5 13.25 13.0 13.25 13.5 13.0 13.25 13.0 13.25 12.75 12.75 Part A How many dots should go above each data value in the line plot? Type a number in each box. Lengths of Roses (inches) 12.75 13.0 13.25 13.5 13.75 14.0 dot(s) above 12.75 dot(s) above 13.0 Review progress Question 7 of С​

Answers

Answer:

2: 12.75

3: 13.0

4: 13.25

3: 13.5

0: 13.75

0: 14.0

The following system of inequalities shows the relationship between two numbers, where the value of x is less than the value of y,
and both numbers are integers.


Which solution is valid within the context of the situation?

Answers

Answer:

(-1,5)

Step-by-step explanation:

A square pyramid has a base with a side length of 6 inches and lateral faces with heights of 11 inches. What is the surface area of the pyramid?

Answers

Answer:

[tex]\huge\boxed{\sf 168\ in.\²}[/tex]

Step-by-step explanation:

Surface area of the base:

= Length * Length

= 6 * 6

= 36 in.²

Surface Area of 1 Lateral Face:

= [tex]\sf \frac{1}{2} (Base * Height)[/tex]

Base = 6 inches

Height = 11 inches

= 1/2 (6*11)

= 3 * 11

= 33 in.²

Surface Area of 4 lateral faces:

= 33 * 4

= 132 in.²

Surface Area of the whole pyramid:

= 132 + 36

= 168 in.²

[tex]\rule[225]{225}{2}[/tex]

Hope this helped!

~AH1807

Answer:

SA = 168 in.²

Step-by-step explanation:

The surface area is the sum of the area of the base and the areas of the 4 congruent triangular sides.

s = side of the base

h = height of each side

SA = s² + 4(sh/2)

SA = (6 in.)² + 4(6 in.)(11 in.)/2

SA = 36 in.² + 132 in.²

SA = 168 in.²

C = ?
when d = 25 m.
(C = circumference)
(d = diameter)

Answers

Answer:

12.5m

Step-by-step explanation:

I don't wanna type it lmco (laughing my cheeks off)

Write the fraction 1/5 as a percent.

Answers

Answer:

20%

Step-by-step explanation:

what is ( 5 x 1 ) + ( 5 x 1/3)

Answers

Answer:

The answer is 20/3

Step-by-step explanation:

(5 x 1 ) + ( 5 x 1/3)

5 + ( 5 × 1/3 )

5 + 5/3 = 20/3

Whats 6 1/2 - 5 5/6 plz help thx

Answers

Answer:

2/3

Step-by-step explanation:

1/2 = 3/6

6 - 5 = 1

1 3/6 - 5/6 = 4/6

4/6 = 2/3

Hope this helped! Have a nice day! Plz mark as brainliest!!!

-XxDeathshotxX

Answer:

Answer is -1.666

Step-by-step explanation:

6*1/2=3

5*5/6=4.1666

The workers' union at a particular university is quite strong. About 96% of all workers employed by the university belong to the workers' union. Recently, the workers went on strike, and now a local TV station plans to interview workers (chosen at random) at the university to get their opinions on the strike. What is the probability that exactly of the workers interviewed are union members

Answers

Answer:

The probability that exaclt x of the members are union members from the n interviewed is given by:

[tex]P(X = x) = C_{n,x}.(0.96)^{x}.(0.04)^{n-x}[/tex]

Step-by-step explanation:

For each worker, there are only two possible outcomes. Either they are union members, or they are not. Members are independent of other members. This means that we use the binomial probability distribution to solve this question.

Binomial probability distribution

The binomial probability is the probability of exactly x successes on n repeated trials, and X can only have two outcomes.

[tex]P(X = x) = C_{n,x}.p^{x}.(1-p)^{n-x}[/tex]

In which [tex]C_{n,x}[/tex] is the number of different combinations of x objects from a set of n elements, given by the following formula.

[tex]C_{n,x} = \frac{n!}{x!(n-x)!}[/tex]

And p is the probability of X happening.

About 96% of all workers employed by the university belong to the workers' union.

This means that [tex]p = 0.96[/tex]

What is the probability that exactly of the workers interviewed are union members

We want probability of x, from n workers interviewed. So

[tex]P(X = x) = C_{n,x}.p^{x}.(1-p)^{n-x}[/tex]

[tex]P(X = x) = C_{n,x}.(0.96)^{x}.(0.04)^{n-x}[/tex]

Which of the following is NOT a requirement of the Combinations​ Rule, ​, for items that are all​ different? Choose the correct answer below. A. That order is taken into account​ (consider rearrangements of the same items to be different​ sequences). B. That r of the n items are selected​ (without replacement). C. That there be n different items available. D. That order is not taken into account​ (consider rearrangements of the same items to be the​ same).

Answers

Answer: A. That order is taken into account​ (consider rearrangements of the same items to be different​ sequences).

Step-by-step explanation:

The combination is usually given by the formula below:

C n,x = n! / x! (n-x)!

We should note that the requirements for a combination rule include:

• That r of the n items are selected​ (without replacement).

• That there be n different items available.

• That order is not taken into account​ (consider rearrangements of the same items to be the​ same).

Therefore, the option that is not a requirement of the Combinations​ Rule, ​, for the items that are all​ different will then be option A"That order is taken into account​ (consider rearrangements of the same items to be different​ sequences).

In ΔJKL, j = 520 inches, k = 730 inches and l=640 inches. Find the measure of ∠K to the nearest degree.

Answers

Answer:

It’s 56.7

Step-by-step explanation:

Fudisks

Question 4 (1 point) The half life of Cs-137 is 30.2 years. If the initial mass of the sample is 100kg, how much will remain after 151 years? Leave your answer as a decimal and do not round, Answer = kg Blank 1:​

Answers

Answer:

3.12 kg.

Step-by-step explanation:

Mass after t years:

The mass of the elements after t years is given by the following equation:

[tex]M(t) = M(0)(1-r)^t[/tex]

In which M(0) is the initial mass and r is the decay rate, as a decimal.

The half life of Cs-137 is 30.2 years.

This means that:

[tex]M(30.2) = 0.5M(0)[/tex]

We use this to find r.

[tex]M(t) = M(0)(1-r)^t[/tex]

[tex]0.5 = M(0)(1-r)^{30.2}[/tex]

[tex](1-r)^{30.2} = 0.5[/tex]

[tex]\sqrt[30.2]{(1-r)^{30.2}} = \sqrt[30.2]{0.5}[/tex]

[tex]1 - r = 0.5^{\frac{1}{30.2}}[/tex]

[tex]1 - r = 0.9773[/tex]

So

[tex]M(t) = M(0)(0.9773)^{t}[/tex]

If the initial mass of the sample is 100kg, how much will remain after 151 years?

This is M(151), with M(0) = 100. So

[tex]M(t) = 100(0.9773)^{t}[/tex]

[tex]M(151) = 100(0.9773)^{151} = 3.12[/tex]

The answer is 3.12 kg.

100 points need help for unit test please!!! don't answer if you don't know
question attached

Answers

To find the x-intercept, substitute in 0 for y and solve for x.

To find the y-intercept, substitute in 0 for x and solve for y.

x-intercept(s): (4,0)

y-intercept(s): (0,−28/5)

Please Mark brainliest and I hope this helps! :D

Answer:

Solution given:

when

(-3,-2)

in equation;

7×-3-5×-2=28

-11≠28(not satisfied)

when

(-2,-3)

equation will be

7×-2+5×3=28

1≠28(not satisfied)

when

(0,4)

equation will be:

7×0-5×4=28

-20≠28

(not satisfied)

when

(4,0).equation will be

7×4-5×0=28

28=28(satisfied)

so

(4,0) is a required answer.

The captain of the basketball team wants to determine where the team should go for their end-of-season celebration. Which of the following is a representative sample? Select all that apply.
A.


The students in her homeroom
B.


Players whose names are randomly selected from a hat
C.


The players in the starting lineup
D.


All players over 5 feet tall
E.


Every 5th player selected from an alphabetical roster of the team

Answers

The answer is c . Players whose names are randomly selected from a hat

The representative samples are Players whose names are randomly selected from a hat and Every 5th player selected from an alphabetical roster of the team. (Option B and E).

What is Representative Sample?

A representative sample is a subset of a larger population that accurately reflects the characteristics, diversity, and distribution of the entire population. When conducting research or making decisions based on data, it is often impractical or impossible to collect information from every individual in the population. Instead, researchers or decision-makers select a representative sample to draw conclusions or make inferences about the entire population.

For a sample to be considered representative, it should be selected in a way that ensures each member of the population has an equal or proportional chance of being included. In the given problem, both options provide a fair chance for each team member to be included in the selection for the end-of-season celebration destination.

Learn more about representative sample here: https://brainly.com/question/30490266

#SPJ2

The original cost is $25. The item has a discount of 20% off. What is the discount amount?

Answers

Answer:20 dollars.

not sure

Step-by-step explanation:

cuanto es 8/9 x 15/16? porfaAa estoy en prueba

Answers

Answer:

=5/6

Step-by-step explanation:

happy to help ya :)

Other Questions
After mixing for three hours, the product is extracted into dichloromethane and the solvent is removed to give 245 mg of an oil. Using the moles of our protected aldehyde calculated earlier (2.96) and the molecular weight of the product (102 g/mol) predict the theoretical 100% yield of the product in milligrams. Round to the tenths place. I need help with this guiding reading assignment Titanium(IV) chloride decomposes to form titanium and chlorine, like this:_____. TiCl4(1)-Ti(s) + 2 Cl 2(g) At a certain temperature, a chemist finds that a reaction vessel containing a mixture of titanium(IV) chloride, titanium, and chlorine at equilibrium has the following composition:_____. compound amount TiCl4 4.18g Ti 1.32g Cl2 1.08gCalculate the value of the equilibrium constant for this reaction. Round your answer to significant digits. Clears your work. Undoes your last action. Provides information about entering answers. Que celula est compuesta por tres partes importantes que son la membrana celular, el citoplasma y el ncleo celula? I just need help on problem B Find the percent of increase from 36 inches to 43 inches. Round to the nearest tenth of a percent if necessary. The function f(x) goes through the point (-2, 6). Which of the following translations of f(x) will go through the point (3, 5)? Describe how the kidney and bladder work together to remove waste from the body. El lado de un cuadrado mide 20 cm. Calcula el incrememto aproximado del area si su lado incrementa 0.1 cm. A toy store needs to make 800 toys to sell during the holidays. Three hundred seventy-two toys have already been made. By late afternoon, 159 more toys were made. How many more toy need to be made so the store will be ready for the holidays? According to a recent poll on the Prevention magazine Web "Most people (56%) reported that even though they are overweight, their doctor never discusses the role of weight in leading a healthy life. Only 14% indicated that it was not an issue because they are already in good shape." The Prevention magazine survey results are provided using _____ to present the statistics. 1. Why does the body need nutrients?2. What are some factors that influence which foods you chose?3. Describe how your emotions can affect your food choices?4. What is the difference between appetite and hunger?5. List the six major classes of nutrients6. Explain with details how the body uses nutrients Can someone please help f(x) = (x + 1)2, g(x) = 2x3One way to find (f g)(x) when x = 2 is to start by composing f(g(x)).Find f(g(x)).4x6 + 14x9 + 14x6 + 4x3 + 14x9 + 4x6 + 1 In the figure below, line a || line b If m Put the following equation of a line into slope-intercept form, simplifying all fractions. The day was scorching hot and bone dry, the air smelled like burnt toast, and everyones nerves were on edge. Only Zeke was calm. He patrolled the perimeter, making jokes about..Continue the story plz Any note that isn't part of the melody is part of the _____. What is the degree measure of Angle P? Anyone know the answer thanks